Verifying a simple QFT derivation in Peskin's

In summary: The vacuum is indeed observer dependent. I don't know where you get your formulae from, but the correct one is U(\Lambda)|0> = |0>, and this is true for all inertial observers.In summary, the conversation is discussing the derivation of equation (2.38) from equations (2.35) and (2.37) in the textbook on QFT. The speaker is asking for help with the derivation, and another participant responds with the steps to take in order to get from (2.35) to (2.38). However, there is a discussion about the validity of the equation and its implications for non-inertial observers. In the end, it is concluded that the existence of
  • #1
infiniteen
2
0
Hiya,

just stumbled upon this forum searching out 'Peskin errata' when trying to figure out a simple QFT calculation in the textbook. Apparently, there is no mention of the simple derivation that I'm struggling with, so there must be something wrong with my own working. I would really appreciate any help with this.

Anyway, here's the derivation -

basically, the I'd like to ask how (2.38) results from (2.35) and (2.37).

[tex]|\textbf{p}\rangle = \sqrt{2E_{\textbf{p}}}a^{t}_{\textbf{p}}|0\rangle[/tex]
(2.35)​

[tex]U(\Lambda)|\textbf{p}\rangle = |\Lambda\textbf{p}\rangle[/tex]
(2.37)​

[tex]U(\Lambda)a^{t}_{\textbf{p}}U^{-1}(\Lambda) = \sqrt{\frac{E_{\Lambda\textbf{p}}}{E_{\textbf{p}}}}a^{t}_{\Lambda\textbf{p}}[/tex]
(2.38)​

It looks like something that you could hardly go wrong with, but I get the following instead:

[tex]U(\Lambda)a^{t}_{\textbf{p}} = \sqrt{\frac{E_{\Lambda\textbf{p}}}{E_{\textbf{p}}}}a^{t}_{\Lambda\textbf{p}}[/tex]

Thanks in advance for any help rendered :)
 
Physics news on Phys.org
  • #2
infiniteen said:
basically, the I'd like to ask how (2.38) results from (2.35) and (2.37).

[tex]|\textbf{p}\rangle = \sqrt{2E_{\textbf{p}}}a^{t}_{\textbf{p}}|0\rangle[/tex]
(2.35)​


Hi,

Insert [itex]U^{-1}U[/itex] between [itex]a^{t}[/itex] and [itex]|0\rangle[/itex] and use the fact that the vacuum is Lorentz invariant;

[tex]U |0\rangle = |0\rangle[/tex]

regards

sam
 
  • #3
You can't actually get from (2.35) to (2.38), and P&S don't claim to. They are using the general framework of unitary transformations in QM. All QM statements (eg, commutation relations, eigenvalue equations, etc) are unchanged if every operator [itex]A[/itex] is replaced by [itex]U\!\!AU^{-1}[/itex], and every state [itex]|\psi\rangle[/itex] by [itex]U|\psi\rangle[/itex], where [itex]U[/itex] is a unitary operator.
 
  • #4
But where does [tex]U(\Lambda)a^{t}_{\textbf{p}}U^{-1}(\Lambda) = \sqrt{\frac{E_{\Lambda\textbf{p}}}{E_{\textbf{p}}} }a^{t}_{\Lambda\textbf{p}} [/tex] come from?
 
  • #5
P&S are simply postulating the existence of such a unitary operator, since symmetries in QM are, in general, implemented by unitary operators. (Find a mind-numbingly detailed exposition of this, see Weinberg volume 1.)
 
  • #6
samalkhaiat said:
Hi,

Insert [itex]U^{-1}U[/itex] between [itex]a^{t}[/itex] and [itex]|0\rangle[/itex] and use the fact that the vacuum is Lorentz invariant;

[tex]U |0\rangle = |0\rangle[/tex]

regards

sam

Isn't this true only for non-accelerating observers?
 
  • #7
haushofer said:
samalkhaiat said:
Isn't this true only for non-accelerating observers?


I was referring to the Unruh effect, or am I talking nonsense now?
 
  • #8
haushofer said:
Isn't this true only for non-accelerating observers?

The vacuum is rotation invariant and boost invariant; rotate it or boost it, and you get the same state back again. That's the meaning of [itex]U(\Lambda)|0\rangle=|0\rangle[/itex].
 
  • #9
Avodyne said:
The vacuum is rotation invariant and boost invariant; rotate it or boost it, and you get the same state back again. That's the meaning of [itex]U(\Lambda)|0\rangle=|0\rangle[/itex].

Yes, I can understand that for inertial observers. But according to the Unruh-effect an accelerating observer will measure that the vacuum |0> as observed by inertial observers will be a heath bath with a certain temperature T. So, as far as I can understand, the statement

U|0> = |0>

only is true for those U which consists of Lorentz transformations transforming inertial observers to inertial observers.

Am I getting something wrong?
 
  • #10
haushofer said:
Yes, I can understand that for inertial observers. But according to the Unruh-effect an accelerating observer will measure that the vacuum |0> as observed by inertial observers will be a heath bath with a certain temperature T. So, as far as I can understand, the statement

U|0> = |0>

only is true for those U which consists of Lorentz transformations transforming inertial observers to inertial observers.

Am I getting something wrong?

The existence of a unique, Poincare' invariant vacuum and its cyclicity is one of the postulates of the (special) relativistic quantum field theory.
I do understand your concerns regarding non-inertial observers and their choices of vacuum! However, I cannot produce a "short" and "easy" answer in here. A very readable account on the issues involved can be found in Sec. 3.3 of Birrell & Davies book "Quantum Fields in Curved Space".


regards

sam
 
  • #11
haushofer said:
[...] according to the Unruh-effect an accelerating observer will measure that the vacuum |0> as observed by inertial observers will be a heath bath with a certain temperature T. So, as far as I can understand, the statement

U|0> = |0>

only is true for those U which consists of Lorentz transformations transforming inertial observers to inertial observers.
That's correct. The Hilbert/Fock space of standard QFT is constructed to carry a
representation of the Poincare group. If one then tries (naively) to represent a larger
group thereon (eg a group also containing accelerations) one gets a new Fock space which
is disjoint from the original. (Well, that's what the maths says anyway. Whether this is an
experimentally-real physical effect remains to be seen.)
 

1. What is the purpose of verifying a simple QFT derivation in Peskin's?

The purpose of verifying a simple QFT derivation in Peskin's is to ensure the accuracy and validity of the derived equations and results. This is an important step in the scientific process to confirm the reliability of the theory and its predictions.

2. What are the steps involved in verifying a simple QFT derivation in Peskin's?

The first step is to carefully analyze the derivation and understand its assumptions and mathematical manipulations. Then, the derivation must be reproduced and compared to the original to check for any discrepancies. Any errors or inconsistencies must be identified and corrected. Finally, the results must be tested against experimental data or other established theories for further validation.

3. Why is it important to verify a simple QFT derivation in Peskin's?

Verifying a simple QFT derivation in Peskin's is important because it ensures the accuracy and reliability of the derived equations and results. It also helps to identify any errors or inconsistencies that may have been overlooked, which can impact the validity of the theory and its predictions.

4. What are the potential challenges in verifying a simple QFT derivation in Peskin's?

Some potential challenges in verifying a simple QFT derivation in Peskin's include complex mathematical manipulations and assumptions, which can be difficult to understand and reproduce. Additionally, there may be discrepancies between the original derivation and the reproduced one, which can take time and effort to identify and correct.

5. How can the results of a verified simple QFT derivation in Peskin's be used?

The results of a verified simple QFT derivation in Peskin's can be used as a reliable basis for further research and theoretical developments in the field of quantum field theory. They can also be used to make predictions and test against experimental data, providing insights into the fundamental nature of particles and their interactions.

Similar threads

  • Quantum Physics
Replies
1
Views
776
Replies
1
Views
962
  • Quantum Physics
Replies
4
Views
2K
Replies
5
Views
2K
Replies
1
Views
2K
  • Quantum Interpretations and Foundations
2
Replies
47
Views
1K
Replies
3
Views
783
Replies
9
Views
3K
  • High Energy, Nuclear, Particle Physics
Replies
5
Views
3K
  • Advanced Physics Homework Help
Replies
1
Views
2K
Back
Top